This is a resolve reconcile explain question, though it may be difficult to identify. The phenomenon we are trying to explain is how the statistic could be accurate if the conclusion isn’t, i.e. how could emergency room visits grow if heroin use remained constant or declined. The question stem asks us: Which one of the following, if true, would account for the statistic above without supporting the author’s conclusion?
The statistic mentioned in the stem is that the amount of emergency room visits by heroin users increased by 20% during the 80s. The author concludes that this was a result of an increase in the use of heroin, but we want to explain why the phenomenon identified by the statistic occurred without supporting this conclusion, so our hypothesis can’t involve an increase in heroin use. We are therefore looking for an alternative hypothesis. Let’s see what we get in the answer choices.
Correct Answer Choice (A) Bingo! This answer gives us a reason for why more heroin users would end up in the hospital even if there wasn’t an increase in heroin use, namely, increased violence from the drug trade.
Answer Choice (B) If it reduced the risk of infection, we would expect a decrease in the number of hospital visits.
Answer Choice (C) Interesting, but this does nothing to explain why there was such an increase in the first place.
Answer Choice (D) Remember, we don’t want to support the conclusion that heroin use increased!
Answer Choice (E) This gives us a more detailed explanation of what specific issues caused heroin users to go to the emergency room, but not why the amount of them who did so increased. This is a more complete description of the phenomenon, not a hypothesis for why it occurred.
This is a resolve reconcile explain question, as the question stem asks: Which one of the following, if true, is most helpful in resolving the apparent paradox?
The stimulus opens with a study by the government, which discovered that consumers who opted for bottled water were receiving a more expensive and dangerous product than the public water supply. Seems like a bad deal! Weirdly, even though the study received a lot of attention, people have been buying even more bottled water. The question stem asks us to resolve a paradox; in this case the paradox is that the opposite outcome we would have expected followed the release of the study. Our job is to select the answer choice which provides the best hypothesis for why this weird result occurred. Let’s see what we get:
Answer Choice (A) This seems like another reason for why people should be choosing drinking water instead of bottled water, and does nothing to explain the fact that they aren’t.
Answer Choice (B) This eliminates a potential difference between the two water sources which might explain the consumer preference, and therefore makes the result even weirder without explaining it at all.
Correct Answer Choice (C) This answer explains the paradox by differentiating between subsets of bottled water. Although many kinds of bottled water were less safe than public water, the increase in sales actually came from specifically the brands that were identified as superior to public water in the report.
Answer Choice (D) This makes it slightly less weird, but still does nothing to explain why there was an increase at all.
Answer Choice (E) We are talking about bottled water, not food, and regardless this suggests that consumers do care about what the government says, which makes their apparent disregard of the study stranger.
This is a resolve, reconcile, or explain question, since the stem asks us: Which one of the following, if true, most helps to explain the study’s finding?
The stimulus begins with a yearlong study, in which half of the participants were given a kit to test their cholesterol. The result of the study was that the participants with testing kits reduced, on average, their cholesterol levels 15% more than the no-kit group. Interesting! So being able to test your cholesterol levels seems to help reduce them. The stimulus finishes with two final tidbits; the participants were (i) randomly selected, and specifically from (ii) those who have very high cholesterol levels. Our job here is to explain why having access to testing kits would enable high-cholesterol individuals to reduce their cholesterol more. We are looking for a powerful hypothesis for the difference between the kit-havers and kit-not-havers which relies on as few assumptions as possible. Let’s see what we get:
Answer Choice (A) All participants had high cholesterol, and it’s unclear how accuracy would even affect the amount participants would reduce their cholesterol.
Answer Choice (B) This would make things even more confusing, because they would have lowered their cholesterol by more while avoiding cholesterol-lowering food!
Answer Choice (C) Ok? This gives us no reason for why the tests themselves helped people lower their cholesterol levels.
Answer Choice (D) Our phenomenon is a difference between the kit-havers and kit-not-havers; this information does nothing to explain that difference among the subsets.
Correct Answer Choice (E) If using the kit reinforced cholesterol reduction efforts, then it would make sense why kit-havers reduced their cholesterol by a greater amount!
This is a must be true question, as the question stem demands: If the statements above are all true, which one of the following must also be true*?*
The first sentence tells us that Angela will earn her Psychology degree once she completes two courses; one in experimental design, the other in developmental psychology. Unfortunately for Angela, experimental design is a pre-requisite of developmental psychology, and won’t be available till the following term, so Angela won’t finish her degree for at least two more terms. The first sentence tells us that the two courses are enough (i.e. sufficient) for Angela to get her degree. We should infer from the second sentence that they are also required (i.e. necessary) for Angela to get her degree, and that ED is required for DP. It’s important for us to recognize that Angela getting a psychology degree and Angela completing developmental psychology are both sufficient and necessary for each other, and that what we have is therefore a biconditional relationship between the two events. If we were to translate these requirements into a conditional chain, it would be something like: P Degree ←→ DP → ED. Since this is all the information the stimulus has for us, we should expect the correct answer to be something guaranteed by these combined conditionals. Let’s see what the answer choices have in store for us:
Answer Choice (A) We’ve been told nothing about how long the course will take.
Answer Choice (B) We’ve been told nothing about the relative difficulty of the two courses.
Answer Choice (C) We’ve been told nothing about the pre-requisites for experimental design, just that it is required for Angela’s degree and to take developmental psychology.
Answer Choice (D) We’ve been told about the requirements for Angela to get a degree, but we can’t infer that is true of anyone seeking a psychology degree in her university. Maybe there are several options to receive the degree, but Angela’s course selection has committed her to needing these two credits specifically.
Correct Answer Choice (E) Angela’s degree needs both DP and ED, but since DP already requires that she have completed ED, then she must have satisfied all the requirements for her degree if she completes DP.
We should recognize this as a must be true question, as the question stem states: If the statements above are true, which one of the following must also be true*?*
This stimulus is quite short for a MBT question. The first sentence tells us about the plan of an airport designed for private aircraft to cover its expenses by charging fees to private aircraft users. Unfortunately, the plan failed and the airport was unable to meet its expenses because the revenue from the fees was lower than expected. And that’s all we get! Since this is a MBT question, we know the correct answer is going to be a 100% guaranteed inference from just this information. The only thing we’ve really been told about is that the expenses were greater than both total revenue (superset) and specifically revenue from user fees (subset). The correct answer will have to somehow relate to this information. Let’s see what we get:
Answer Choice (A) We’ve been told nothing about where the county’s citizens live!
Answer Choice (B) The fact that there was any revenue from user fees strongly suggests this is at least partly false, and it certainly isn’t something we know for sure.
Answer Choice (C) This is just a total non sequitur, nothing was said about the airport’s construction.
Correct Answer Choice (D) If the airport was unable to pay its operating expenses, then its expenses must have been larger than its total revenue, and therefore it must be true that its expenses were greater than a subset of its total revenue.
Answer Choice (E) This requires a lot of assumptions as we know very little about the airport’s users.
We know this is a sufficient assumption question because of the question stem: “which one… if assumed… would allow [the author] to draw her properly to draw her conclusion…” Interestingly, the conclusion is given to you in the stem. This should help in our formal analysis to figure out what the conclusion is.
Sufficient assumption questions tend to be very formal. We’re looking for a rule that would validate the conclusion, specifically by bridging the premise and conclusion through the rule. Not only are we extrapolating the rule from our argument, but we’re also using that rule to render the argument “valid.” The way to prephrase our answer choice is by tying our premises and conclusion together into a rule: “If [premise] → then [conclusion].”
Our first sentence tells us that an antitheft alarm may stop an attempted theft at night on a crowded street. I’m imagining an incredibly loud siren going off in a middle of a downtown area... I think it’s possible that theft could be prevented with a loud alarm like that.
The next sentence gives an alternative cause for the alarm going off: instead of a thief, it could be a branch or another form of contact.
The third sentence puts aside the causes and says in any of the situations in which the alarm goes off at night, it’s disturbing the sleep of people in the neighborhood. Makes sense - a blaring alarm would definitely disturb my sleep!
Our last sentence, the conclusion which we read in our stem, is: out of consideration for our neighbors, we should deactivate the alarms when parking in crowded city neighborhoods at night.
Why should they?? Protecting a car is probably more important and valuable to its owner than someone else’s sleep, and there doesn’t seem to be a moral code that says people should deactivate their cars if it bothers people’s sleep. On the chance that the car is getting stolen, an alarm would be the best thing to help avoid losing the car! This prescriptive conclusion is assuming that others’ sleep is more important than protecting someone’s car.
What we need is a rule that bridges our premises to our conclusion and validates this prescription. Our premise here is that if the alarm goes off, then people’s sleep is disturbed, and we should be considerate of this. Our conclusion is that owners of these cars should deactivate their alarms. Put them together to make our rule! If people’s sleep is getting disturbed and we should be considerate of this → owners should deactivate car alarms at night.
Answer Choice (A) This would weaken the argument! This is putting the protection of peoples’ cars over peoples’ sleep.
Answer Choice (B) In most cases? What about some cases in which it’s actually a theft? And besides this, the answer choice still does not justify why we should deactivate our alarms. Just because it is a false alarm, it does not validate the argument.
Correct Answer Choice (C) While it’s not a perfect paraphrase of our rule, it gets to the idea that sleep is more important than protecting cars.
Answer Choice (D) Remember, our conclusion is prescriptive. How does saying “people who have alarms are inconsiderate” help our argument? Does this mean they should deactivate their alarms? No – this is out.
Answer Choice (E) We don’t really care about what happens to the alarms during the day, we’re concerned with why people should turn alarms off at night. This is supplemental information that does nothing for our stimulus.
This is a most strongly supported question, as the question stem asks: If both Monroe’s conclusion and the evidence on which he bases it are correct, they would provide the strongest support for which one of the following?
From the question stem alone we already know someone named Monroe is going to be giving us a conclusion supported by some evidence. The first sentence gives us a phenomenon; Monroe ate at the Tip-Top Restaurant and enjoyed the meals, but afterwards became ill each time. He must have really enjoyed it to go back after being sick two times in a row! Since we have started with a phenomenon, Monroe getting sick every time he eats at Tip-Top, we should expect a causal hypothesis as to why this correlation is occurring. The next sentence gives us some more detail about this correlation. Monroe ate three different meals at Tip-Top, but each one had hot peppers. So this Monroe getting sick every time he eats at Tip-Top correlation is also more specifically a Monroe getting sick every time he eats hot peppers correlation. Where the first sentence probably made you think, “Maybe you should try a new restaurant, Monroe!”, this new information has properly altered your hypothesis to, “Maybe ask them to skip the hot peppers next time, Monroe!”. The final sentence tells us that Monroe agrees and gives us his conclusion, his hypothesis for why he kept getting sick; it was the hot peppers causing him to feel sick after all three meals. He probably should have been able to figure that out after two meals, but good for Monroe!
Having now read the stimulus, and remembering that we are supposed to assume Monroe’s hot pepper hypothesis is correct, our job is to select the answer that is most strongly supported based off of it. Let’s take a look at the answer choices:
Answer Choice (A) What this answer choice does is confuse sufficiency with necessity. Just because hot peppers are enough for Monroe to feel sick, doesn’t mean they are required for him to be sick. Maybe he’s also allergic to pine nuts and the all-you-can-eat specials includes a really nice Pesto pasta! If you chose this answer, you might want to review conditional logic.
Correct Answer Choice (B) This is the correct answer because it does what A tries to do without its error of confusing enough with required. Where A said Monroe can eat anything without hot peppers and not get sick, B correctly infers that if Monroe ate a different dish with hot peppers he would have still gotten sick, because hot peppers are enough for him to be ill.
Answer Choice (C) Same issue as A, this question incorrectly assumes that hot peppers are the only possible cause of Monroe feeling ill.
Answer Choice (D) We need to assume that we just don’t know about Monroe’s eating history for this answer to follow from our stimulus.
Answer Choice (E) This should be clearly wrong. We know nothing about Monroe’s restaurant history outside of Tip-Top. Maybe Monroe is really bad at noticing when an ingredient makes him feel sick and he’s actually eaten hot peppers 20 times at other restaurants and gotten sick, and only now after three meals at Tip-Top made the connection.
This is a most strongly supported question, since the question stem says: Which one of the following conclusions is most strongly supported by the results of the experiments?
As you should have noticed in the question stem, this stimulus involves experiments. We should therefore be on the lookout for a phenomenon. The first sentence begins with the conditional indicator only, and from it we learn that some strains of tobacco are resistant to tobacco mosaic virus; important to note that resistant here doesn’t mean they resist infection, but rather they resist developing symptoms and becoming diseased. This resistance is our phenomenon. The first of the experiments gives us a correlation. Tobacco plants infected with the mosaic virus who resisted its effects had increased levels of salicylic acid, while non-resistant plants had no increase in their levels. Resistance to the virus correlates with more salicylic acid. The second experiment greatly strengthens the correlation by artificially raising the salicylic levels of half of 50 non-resistant plant, with every high salicylic level plant resisting the disease and every normal plan succumbing to it.
If we were to summarize this stimulus, it is essentially: phenomenon, correlation, even stronger correlation. What we never receive in this stimulus is a conclusion, or more specifically in this case, a hypothesis. If we pre-phrase, we would expect a conclusion to this information to infer causation from correlation, and conclude that the increased salicylic acid at least partly causes the mosaic resistance. We should be on the lookout for an answer that introduces this hypothesis, while eliminating any answers that have little to no support in the stimulus. Let’s take a look at our options:
Answer Choice (A) None of the experiments involved salicylic acid removing symptoms, but only preventing them. Our correlation is between salicylic acid and resistance, not healing.
Correct Answer Choice (B) This answer does exactly what we should have predicted in our pre-phrasing; it introduces the expected hypothesis that salicylic acid at least partly explains why some plants are able to resist the mosaic virus.
Answer Choice (C) This information is not only not supported, but likely false based on the stimulus. The salicylic acid is described as naturally occurring in the plants, and the difference between the resistant and non-resistant plants is whether there was an increase in salicylic acid.
Answer Choice (D) Remember we are told about an increase in the salicylic levels following infection; if the plant is uninfected the stimulus suggests that its salicylic acid levels will be normal, and maybe even indistinguishable from those of non-resistant plants.
Answer Choice (E) This answer might be appealing since it does emphasize the correlation between an increase in salicylic acid and resistance to the virus. The problem is that the experiments involved artificially injecting plants to raise their salicylic levels, and we have no information about whether it is possible increase the plant’s production of the acid.
This is a weakening question, since we are asked: Which one of the following, if true, most seriously weakens the argument?
This stimulus is relatively straightforward. We are first told that a portion of economically useful raw materials are both nonrenewable and in limited supply on Earth. So we can’t make more of these resources and we don’t have such a big supply that we would never run out. Following this premise we get a sentence beginning with the conclusion indicator therefore and the conditional indicator unless. If we translate this conditional conclusion, we should end up with something along the lines of: no materials can be obtained outside earth → unable to accomplish what is now accomplished with those materials (referential phrase: the useful, non-renewable, and limited supply materials).
When we are faced with a weakening question involving a standard argument, we should always consider if the argument relies on a large assumption. In this case, we are concluding that if we can’t find more of these materials, we won’t be able to do what we use them for right now. The major assumption here is that these materials are required for what we do with them. For example, just because we currently use gas to produce electricity doesn’t mean that gas is required to produce electricity. An easy way to weaken this argument is to explicitly state that these materials aren’t required, and could be replaced with alternatives. We should be on the lookout for an answer choice that does this. Let’s see what we get:
Answer Choice (A) The argument we are trying to weaken is about the portion of economically useful materials which aren’t renewable. Even if there are renewable resources, unless they are viable alternatives to our non-renewables, we don’t care about them.
Answer Choice (B) This answer choice just suggests that the conditional conclusion of our argument will be triggered, since it is hard to get resources outside of earth. This does nothing to weaken or even strengthen the argument.
Correct Answer Choice (C) This contradicts the assumption we identified in the stimulus. If these limited nonrenewable resources can be replaced with alternatives, then even if we can’t find more of them we can still keep doing what we are doing with them. In short, this says the resources aren’t required, which is exactly what the argument assumes they are.
Answer Choice (D) Who cares? The argument we are trying to weaken has a conditional conclusion; it makes a prediction about what will happen in a particular case. The worthiness of what the resources are being used for is irrelevant to whether it will be impossible to continue should they run out.
Answer Choice (E) This answer fills in one of the assumptions the argument makes; that we will ever run out of these resources just because there is a limited supply of them. For that reason, if anything, all this answer does is strengthen what we want to weaken.
Here we have a flaw question, which we know from the question stem: “The reasoning used by the gallery owner is flawed because it…” Right away we know our correct answer has to do two things: be descriptively accurate, and describe the flaw of the stimulus. We also know what the wrong answers will do - describe reasoning flaws we’ve seen before, but don’t like up with our stimulus. Once we have a clear understanding of the questrion’s objective, we can proceed into structural analysis of the stimulus.
The argument begins by telling us because this certain painting does not appear in Van Gogh’s catalog, we cannot guarantee its authenticity. But our speaker continues to say that because there are such incredible similarities between this painting and Van Gogh’s (brush strokes, color combinations, etc.) it must be the case this is almost certainly an uncatalogued work made by Van Gogh himself.
The conditional reasoning in the stimulus can help reveal the issue with the gallery owner’s argument. Our first sentence says because it's not in the catalog (/C) → there is no guarantee of authenticity (/G). Let’s write out the contrapositive. By negating both variables and switching their positions, we learn that if there is a guarantee of authenticity it must be the case that the art appears in the artist’s catalog because G → C. Our author continues past this premise to incorrectly conclude this particular painting must be a work of the author’s without being able to meet the necessary condition presented in our first sentence.
While the diagrammable relationships are useful, they are not required in order to predict what is wrong with this stimulus. Just because the paintings are similar in style to Van Gogh’s does not mean they came from his catalog. It could very well be the case that a talented amateur wanted to imitate the work of their favorite artist. Knowing that our correct answer choice will point out the weakness in the connection between similar styles to authenticity, we can proceed into answer choice elimination.
Answer Choice (A) This answer choice is descriptively accurate, but not the ultimate issue with our stimulus. Whether or not there is “general agreement” on a topic does not connect to our stimulus concerning a piece of art for which there is actually no apparent general agreement.
Answer Choice (B) Here is another answer choice that descriptively along with the answer choice but does not go far enough in reaching the actual problem in our stimulus. Accusing the argument of failing to cite “expert authority” does not weigh on our discussion about the likelihood a similarity leads us to some level of guarantee of authenticity.
Answer Choice (C) This is not accurate compared to the words of our argument. This answer choice says the only reason in existence for wanting a painting is to make a profit. But we do not have nearly the level of support required in order for us to say the argument makes a conclusion about the sole existing reason for wanting art in the first place.
Correct Answer Choice (D) This is exactly what we are looking for. This descriptively correct answer choice is the only one that correctly points out the more reasonable explanation - we’re looking at a painting that happens to be similar, but very well may not be painted by Van Gogh himself.
Answer Choice (E) Here we have another answer choice that is not descriptively accurate. Our argument does not attempt to push the conclusion on the basis of “self interest” of the reader. Without any connection to the self-interest or benefits included in making these catalog decisions we can eliminate this answer choice.